LSAT and Law School Admissions Forum

Get expert LSAT preparation and law school admissions advice from PowerScore Test Preparation.

 SherryZ
  • Posts: 124
  • Joined: Oct 06, 2013
|
#12765
Dec 2001 LSAT, Sec 3, Q 26:

I don't understand why D is wrong, is it because it actually WEAKENS the argument by saying that the experiments could not be measured precisely??

Thank you so much!

---Sherry
 BethRibet
PowerScore Staff
  • PowerScore Staff
  • Posts: 200
  • Joined: Oct 17, 2012
|
#12793
Hi Sherry,

Thanks for writing in. Yes, your analysis is on target. Answer choice D, if true would weaken, rather than strengthening the argument for a fifth universal force, because it indicates that the experiments which helped to detect that fifth force were not reliable.

Hopefully this helps, but let us know if not!

Beth
 ellyb
  • Posts: 16
  • Joined: Aug 29, 2016
|
#28130
So, as learned from the PS LR bible, there are a number of errors in the use of evidence. One such error is "lack of evidence against a position taken to prove that position is true."

After carefully studying this, I eventually wrapped my brain around the concept and completely concurred that just because there is no proof that something didn't happen, that does not prove that it did indeed happen. So everything was fine for a solid number of months until I stumbled upon this question that threw me for a loop.

[Licensed content removed. The question refers to PrepTest 36, December 2001: LR2, Question #26. - ADMIN]

This unique logical quality - answer choice (B), which is correct - to me seems incompatible with the aforementioned flaw. Since learning that no proof against something doesn't imply it's existence, i'm finding it hard to reconcile the validity of this answer choice. Don't get me wrong, I completely understand in logical terms why - if there was nothing that negates its existence - one might be lead to believe there is a greater chance of its likelihood, however, it seems like the complete antithesis to specifically LSAT logic, especially with that very notable 'flaw in the reasoning' further cementing my aversion to said logic.

Can somebody please explain how, in one instance, no proof against something does not prove its existence.. and to imagine so is a flaw, and yet on the other hand, in some questions, no proof against something does indeed strengthen the likelihood of its existence... and in what respect one can learn to differentiate between the two. Thank you.
 Nikki Siclunov
PowerScore Staff
  • PowerScore Staff
  • Posts: 1362
  • Joined: Aug 02, 2011
|
#28195
Hi ellyb,

Thanks for the question, and welcome to the forum!

You're asking a great question: how do we reconcile the fact that answer choice (B) strengthens the argument with the logical flaw that it seems to entail? Indeed, lack of evidence against a position does not prove the position to be true: for example, just because we cannot disprove the existence of God does not, in and of itself, prove that God exists. Note, however, that the Use of Evidence fallacy is triggered by the unwarranted assumption that lack of evidence against a position (or for a position) proves that the position is true (or false). There is a critical difference between strengthening a position and proving it to be true, just like there is a difference between weakening an argument and proving it to be false. You appear to conflate the two, which is, ironically, a different logical flaw frequently tested on the LSAT. :-D

Re: Question 26 from LR2 of the December 2001 test, the case isn't that the author is arguing that answer choice (B) proves the existence of a fifth universal force. That would indeed be an error in the use of evidence. However, if all of the previously established theories are compatible with the existence of such a force, this makes it more likely that such a force actually exists. Our goal is to strengthen the conclusion, not necessarily prove it to be true: this is a Strengthen, not a Justify, question. By stating that all existing theories are totally cool with the existence of a fifth force, answer choice (B) eliminates a potential problem, namely, that the fifth force might contradict a previously established theory. This eliminates a potential weakness, thereby strengthening the conclusion of the argument.

Hope this clears things up! :) Please let me know if you have any follow-up questions.

Thanks,
 bonnie_a
  • Posts: 32
  • Joined: Jun 05, 2021
|
#91489
I do understand what makes B a correct answer for this question but cannot wrap my head around why D doesn't work. If the experiments were done in settings where factors couldn't be measured with any precision and the possible fifth universal force would explain something about it, then wouldn't that strengthen the argument? Though the experiments themselves were not precise, the fifth force can explain something about it better than the other theory predicted. While I do see how this can also weaken but I felt it could also work the other way around. For example, even if one's work for the math question was wrong, I might see what was done wrong and explain it better than the other person did. Therefore, my explanations still hold and I can be right...?
 Adam Tyson
PowerScore Staff
  • PowerScore Staff
  • Posts: 5153
  • Joined: Apr 14, 2011
|
#91515
Answer D gives us an alternate cause for the problematic measurements in those experiments in which the existing theory did not accurately predict gravitational forces. If the measurements were unreliable, we don't need some mysterious new force to explain them. We can explain them just by saying that they may have been inaccurate!

Consider this, too, bonnie_a: you said that this answer could also weaken the argument. That means that the impact of the answer depends on how we interpret it, right? But if that's the case, then the answer does not, by itself, help the argument. It can only help if we bring in some outside assumptions that are not part of that answer choice, and that is a classic case of a wrong answer! The correct answer must strengthen the argument without our help!

If we want an answer that does the most to strengthen the argument, we don't want any ambiguity, and we need an answer that does that work all by itself, without any need for us to add our assumptions or make any special interpretations. D, on its face, hurts the argument by giving an alternate explanation for the measurements, and by showing that the experiments intended to support the new theory were not reliable. It's both an alternate cause AND a data problem, two common ways to attack a causal relationship.
User avatar
 Sean Kim
  • Posts: 6
  • Joined: Apr 24, 2022
|
#94995
Hello :)
I am curious why (A) is wrong.
If the answer choice went "~was not available to physicists before 1986" instead of "1970s", could it possibly strengthen the position?

Thanks!
 Adam Tyson
PowerScore Staff
  • PowerScore Staff
  • Posts: 5153
  • Joined: Apr 14, 2011
|
#95007
I think that would make answer A more attractive, Sean Kim, but still not good enough to strengthen the argument. That would still require us to make an assumption that the older equipment was unable to measure those small differences, and we shouldn't be helping answer choices by making unwarranted assumptions.

And even if we improved that answer further by saying "we could not have accurately measured this stuff before the experiments that started in 1986," that wouldn't necessarily help the argument in favor of a 5th force, because there still could be plenty of alternate explanations for the difference between the predictions and the results. That answer wouldn't eliminate any alternatives or show, as answer B does, that there are no immediately obvious problems with the theory.

Basically, the instrumentation thing might help to show why we never came up with this theory before, but it doesn't do anything to show that the theory is actually correct.
User avatar
 Sean Kim
  • Posts: 6
  • Joined: Apr 24, 2022
|
#95017
Adam Tyson wrote: Mon Apr 25, 2022 9:44 pm I think that would make answer A more attractive, Sean Kim, but still not good enough to strengthen the argument. That would still require us to make an assumption that the older equipment was unable to measure those small differences, and we shouldn't be helping answer choices by making unwarranted assumptions.

And even if we improved that answer further by saying "we could not have accurately measured this stuff before the experiments that started in 1986," that wouldn't necessarily help the argument in favor of a 5th force, because there still could be plenty of alternate explanations for the difference between the predictions and the results. That answer wouldn't eliminate any alternatives or show, as answer B does, that there are no immediately obvious problems with the theory.

Basically, the instrumentation thing might help to show why we never came up with this theory before, but it doesn't do anything to show that the theory is actually correct.
thank you! It makes much more sense now :)
User avatar
 CJ12345:
  • Posts: 56
  • Joined: May 25, 2023
|
#103260
Hi, Powerscore,
I am still having difficulty being convinced B is right AC. Even though B could help the link between premise and conclusion, B is not strong enough to strengthen the argument. What if previously established scientific results are incompatible with the fifth universal force? It still could not disprove the existence of the fifth universal force. Thus, by stating that no previously established scientific results are incompatible with the notion, the AC does not do much to the argument. Could you help me to explain why my reasoning is not right?

Get the most out of your LSAT Prep Plus subscription.

Analyze and track your performance with our Testing and Analytics Package.